A 0.350m radius solid cylinder is released from rest and rolls down a ramp inclined 21.0° from horizontal. The moment of inertia of a solid cylinder is ½ MR2. After the cylinder has rolled a distance of 5.00m find: the speed of the cylinder

Answers

Answer 1

To find the speed of the cylinder, we can use the conservation of energy principle. The initial potential energy of the cylinder at the top of the ramp is converted to kinetic energy at the bottom of the ramp.

The potential energy of the cylinder is given by mgh, where m is the mass of the cylinder, g is the acceleration due to gravity, and h is the height of the ramp. The height of the ramp is given by h = 5.00m sin(21.0°) = 1.802m.

The kinetic energy of the cylinder is given by ½mv^2, where v is the speed of the cylinder.

Equating the initial potential energy to the final kinetic energy, we have:
mgh = ½mv^2

Substituting the mass of the cylinder and the height of the ramp, we have:
(0.5)(9.81 m/s^2)(0.350m)sin(21.0°) = ½(0.5kg)v^2

Simplifying, we get:

v = √[2(9.81 m/s^2)(0.350m)sin(21.0°)]
v = 2.80 m/s

Therefore, the speed of the cylinder after rolling a distance of 5.00m down the ramp is 2.80 m/s.

To know more about conservation of energy principle:

https://brainly.com/question/16881881

#SPJ11


Related Questions

a nicotine patch on the skin (for people trying to quit smoking) absorbs nicotine directly into the bloodstream. some stronger patches contain 20 mg nicotine. how many patches would kill half of people weighing 60kg?

Answers

Approximately 1.5 nicotine patches containing 20 mg each would be required to potentially be lethal to half of the people weighing 60 kg

To answer your question, we need to calculate the lethal dose of nicotine for a person weighing 60 kg and then find out how many 20 mg nicotine patches would be required to reach that dose.

1. First, we need to know the median lethal dose (LD50) of nicotine, which is the dose that would be lethal to 50% of the population. The LD50 of nicotine is approximately 0.5 to 1 mg per kg of body weight.

2. Now, let's calculate the lethal dose for a 60 kg person:
LD50 = 0.5 mg/kg (minimum lethal dose) x 60 kg = 30 mg (minimum lethal amount)

3. Finally, we will determine how many 20 mg nicotine patches would reach the lethal dose:
Number of patches = Lethal dose / Patch nicotine content = 30 mg / 20 mg/patch = 1.5 patches

It's important to note that individual reactions to nicotine may vary, and this calculation is a rough estimate. In this scenario, approximately 1.5 nicotine patches containing 20 mg each would be required to potentially be lethal to half of the people weighing 60 kg. However, always follow the recommended usage instructions for nicotine patches to ensure safety.

More on nicotine: https://brainly.com/question/24212775

#SPJ11

A machine gear consists of 0.10 kg of iron and 0.16 kg of copper. How much total heat is generated in the part if its temperature increases by 35 C°? (Specific heats of iron and copper are 450 and 390 J/kg×°C, respectively.)

Answers

To calculate the total heat generated in the part, we need to use the formula:

Q = mcΔT

Where Q is the total heat generated, m is the mass of the part, c is the specific heat of the material, and ΔT is the change in temperature.

Given:

Mass of iron (m1) = 0.10 kg
Specific heat of iron (c1) = 450 J/kg°C
Mass of copper (m2) = 0.16 kg
Specific heat of copper (c2) = 390 J/kg°C
Change in temperature (ΔT) = 35°C

To find the total heat generated, we need to calculate the heat generated by each material and then add them together.

Heat generated by iron (Q1) = m1c1ΔT
= 0.10 kg x 450 J/kg°C x 35°C
= 1575 J

Heat generated by copper (Q2) = m2c2ΔT
= 0.16 kg x 390 J/kg°C x 35°C
= 2184 J

Total heat generated (Q) = Q1 + Q2
= 1575 J + 2184 J
= 3759 J

Therefore, the total heat generated in the part is 3759 J.

TO KNOW MORE ABOUT Heat genration CLICK THIS LINK-

brainly.com/question/21959031

#SPJ11

consider an object containing 12 one-dimensional oscillators (this object could represent a model of 4 atoms in an einstein solid). there are 4 quanta of vibrational energy in the object. (a) how many microstates are there, all with the same energy?

Answers

The number of microstates with the same energy is 1365.

The number of one-dimensional oscillators, N = 12

The number of quanta, n = 4

The equation for calculating the number of microstates is given by,

Ω = (n + N- 1)! /n! (N - 1)!

Ω = (4 + 12 -1)! /4! (12 -1)!

Ω = 15!/(4! x 11!)

Ω = 1365

To learn more about microstates, click:

https://brainly.com/question/31493438

#SPJ4

450 C of charge flows through a motor and 9000 J of energy are converted in the motor. 1800 J are dissipated in the cell. The EMF of the cell is:

Answers

The EMF of the cell is 24 volts.

We can use the relationship between charge, energy, and EMF to solve this problem. The energy converted in the motor is equal to the product of the EMF of the cell and the charge that flows through it:

E = EMF * Q

where E is the energy, EMF is the electromotive force of the cell, and Q is the charge.

From the problem statement, we know that 450 C of charge flows through the motor and 9000 J of energy are converted in the motor. We also know that 1800 J of energy are dissipated in the cell. Therefore, the total energy provided by the cell is:

E_total = E_motor + E_dissipated

= 9000 J + 1800 J

= 10800 J

Using the equation above, we can solve for the EMF of the cell:

EMF = E/Q = E_total/Q = 10800 J/450 C = 24 V

Therefore, the EMF of the cell is 24 volts.

To learn more about electromotive visit:

https://brainly.com/question/13753346

#SPJ11

One evening at midnight, you observe Leo high in the southern sky at midnight. Virgo is to the east of Leo and Cancer is to the west. One month earlier, which of these constellations was high in the southern sky in at midnight?

Answers

The one month earlier, Leo would have been high in the southern sky at midnight, with Virgo to the east and Cancer to the west, just as it was on the night  but it would have moved slightly to the west.

As the Earth revolves around the Sun, it also rotates on its axis, which causes the apparent positions of the stars to shift throughout the year. The stars appear to move across the sky due to the Earth's rotation, but they also appear to move over the course of the year due to the Earth's orbit around the Sun.

The constellations that are visible in the southern sky at midnight depend on the season and the location of the observer. In general, the constellations that are visible at a particular time of night will be the same approximately one month later, but they will appear slightly shifted to the west due to the Earth's orbit around the Sun.Leo, Virgo, and Cancer are all zodiac constellations that lie along the ecliptic, which is the apparent path of the Sun across the sky. Leo is located to the east of Cancer and to the west of Virgo along the ecliptic.If Leo is high in the southern sky at midnight, this means that it is on the meridian (an imaginary line running from due south to due north through the zenith) at that time. Virgo would be to the east of Leo, while Cancer would be to the west.One month earlier, the Earth would have moved approximately one twelfth of its way around the Sun in its orbit, which means that the stars would appear to have shifted approximately one twelfth of the way around the celestial sphere. Leo would still be located in the southern sky at midnight, but it would have moved slightly to the west.

for such more questions on celestial sphere

https://brainly.com/question/14839466

#SPJ11

A positively charged rod is brought close to an uncharged electroscope. While the rod is close, one's finger touches the far side of the metal ball on the electroscope. The finger is removed and then the rod is taken away. The electroscope is

Answers

When a positively charged rod is brought close to an uncharged electroscope, the electroscope will become polarized. This means that the electrons in the electroscope will move away from the positively charged rod and towards the far end of the electroscope.

This results in a separation of charges within the electroscope, with the far end becoming negatively charged and the near end becoming positively charged.

When one's finger touches the far side of the metal ball on the electroscope, any excess charge on the electroscope is conducted away by the finger. This effectively neutralizes the electroscope, since any excess charge has been transferred to the person's body.

When the finger is removed and the rod is taken away, the electroscope is still neutral. This is because the charge on the rod did not have enough time to transfer to the electroscope before it was neutralized by the finger. Therefore, the electroscope remains in its original uncharged state.

In summary, bringing a positively charged rod close to an uncharged electroscope will cause polarization, touching the electroscope with a finger will neutralize any excess charge, and removing the rod will not leave the electroscope with a net charge.

For more such questions on Electroscope.

https://brainly.com/question/10470196#

#SPJ11

Which statement about a system of point charges that are fixed in space is necessarily true?
(A) If the potential energy of the system is negative, net positive work by an external agent is required to take the charges in the system back to infinity.
(B) If the potential energy of the system is positive, net positive work is required to bring any new charge not part of the system in from infinity to its final resting location.
(C) If the potential energy of the system is zero, no negative charges are in the configuration.
(D) If the potential energy of the system is negative, net positive work by an external agent was required to assemble the system of charges.
(E) If the potential energy of the system is zero, then there is no electric force anywhere in space on any other charged particle not part of the system.

Answers

The statement that is necessarily true about a system of point charges that are fixed in space is (D) If the potential energy of the system is negative, net positive work by an external agent was required to assemble the system of charges.

This statement is based on the fact that the potential energy of a system of charges is the work required to bring the charges from infinity to their fixed positions. If the potential energy is negative, it means that the charges are attracted to each other and therefore, an external agent had to do work to overcome this attraction and bring the charges together. This is the work required to assemble the system of charges.

Option (A) is incorrect because if the potential energy of the system is negative, it means that the charges are attracting each other and releasing them to infinity would require an external agent to do work.

Option (B) is incorrect because if the potential energy of the system is positive, it means that the charges are repelling each other and bringing a new charge would require work to overcome this repulsion.

Option (C) is incorrect because the potential energy being zero does not necessarily mean that there are no negative charges in the system.

Finally, option (E) is incorrect because the potential energy being zero does not mean that there is no electric force in space on other charged particles not part of the system.

For more such questions on Potential energy.

https://brainly.com/question/20626677#

#SPJ11

A disk can rotate about its central axis like a merry-go-round. Determine the sign of the angular displacement for the following pairs of initial and final angular positions. The initial angular position is -3 rad and the final angular position is +5 rad.

Answers

The angular displacement of a rotating disk, starting at -3 rad and ending at +5 rad with clockwise rotation, is +8 rad.

If the initial angular position is -3 rad and the final angular position is +5 rad, and the direction of rotation is clockwise as viewed from a clock hanging on a vertical wall, then the angular displacement can be calculated as

Angular displacement = Final angular position - Initial angular position

= (+5 rad) - (-3 rad)

= +8 rad

Since the direction of rotation is clockwise, the angular displacement is positive.

Therefore, the sign of the angular displacement is positive (+) for the given pair of initial and final angular positions.

To know more about angular displacement:

https://brainly.com/question/26483834

#SPJ4

--The given question is incomplete, the complete question is given

" A disk can rotate about its central axis like a merry-go-round. Determine the sign of the angular displacement for the following pairs of initial and final angular positions. The initial angular position is -3 rad and the final angular position is +5 rad. the direction of rotation as you view a clock hanging on vertical wall

The bit-rate for a 16-bit stereo sound sampled at 32 kHz is 1.024 Mbps. What is the highest usable audible frequency in the digital media file?

Answers

The highest usable audible frequency in the digital media file is 16 kHz.

What is the highest usable audible frequency in the digital media file?

To find the highest usable audible frequency in a 16-bit stereo sound sampled at 32 kHz with a bit-rate of 1.024 Mbps, you can use the Nyquist theorem. The theorem states that the highest frequency that can be accurately represented is half of the sampling rate.

Identify the sampling rate.
The sampling rate is 32 kHz.

Apply the Nyquist theorem.
The highest usable audible frequency = (Sampling rate) / 2
The highest usable audible frequency = 32 kHz / 2

The digital media file's maximum usable audible frequency is 16 kHz.

Learn more about frequency.

brainly.com/question/5102661

#SPJ11

How much a certain noise will annoy an individual depends on:

Answers

The amount of annoyance caused by a certain noise can vary depending on several factors, including:

Sound level: The loudness of a noise is typically measured in decibels (dB). The higher the sound level, the more likely it is to cause annoyance.

Duration: The longer the duration of a noise, the more likely it is to cause annoyance.

Frequency content: Different frequencies of sound can have different effects on people. Some frequencies are more likely to be perceived as annoying than others.

Context: The context in which the noise occurs can affect how annoying it is perceived to be. For example, a loud noise in a quiet environment may be more annoying than the same noise in a noisy environment.

Personal sensitivity: Individuals vary in their sensitivity to noise. Some people are more easily annoyed by noise than others.

Previous experience: Past experiences with a particular noise or similar noises can influence how annoying it is perceived to be.

Control: The level of control an individual has over the noise can affect their annoyance. For example, if they can turn down the volume of a noisy environment, they may be less annoyed than if they have no control over the noise.

Learn more about certain noise

https://brainly.com/question/30161909

#SPJ4

Suppose that you rub the metal back and forth for twice as long a time. Will the temperature change be different from before? If so, how will the temperature change differ?

Answers

If you rub the metal back and forth for twice as long a time, the temperature change will likely be different from before. Specifically, the temperature change will likely be greater because the metal will have been exposed to more frictional forces and thus more energy will have been transferred to the metal.

When you rub two surfaces together, friction is generated, which can cause the surfaces to heat up. The amount of heat generated depends on several factors, including the materials and roughness of the surfaces, the force applied, and the duration of the rubbing.

When you rub the metal back and forth for a longer time, the metal is exposed to more frictional forces, which results in more energy being transferred to the metal. This increased energy transfer leads to a greater temperature change in the metal, assuming that other factors remain constant.

However, it's important to note that there are limits to how much temperature change can occur due to rubbing. If the rubbing is too intense or prolonged, the metal can become damaged or deformed, which may limit the amount of temperature change that can occur. Additionally, other factors, such as the thermal conductivity of the metal and the surrounding environment, can affect temperature change as well.

In summary, if you rub the metal back and forth for twice as long a time, the temperature change will likely be greater due to increased energy transfer to the metal. However, there are limits to how much temperature change can occur, and other factors can also affect the outcome.

Learn more about temperature here:

https://brainly.com/question/26866637

#SPJ11

A road bike has tires that have a diameter of 0.800m and is rolling down the road at 20.0m/s. Seeing a stop sign the bike rider applies the breaks to bring the bike to a stop in 100.0m. What is the angular displacement of the tires as the bike comes to a stop?

Answers

The angular displacement of the tires as the bike comes to a stop is approximately 251.02 radians.

How to find angular displacement?

The linear displacement of the bike is given by:

d = 100.0 m

The circumference of the tires is given by:

C = πd = π(0.800 m) = 2.51 m

The number of revolutions the tires make as the bike comes to a stop is:

n = d / C = 100.0 m / 2.51 m = 39.84 revolutions.

The angular displacement of the tires is equal to the number of revolutions multiplied by 2π radians per revolution:

θ = n × 2π = 39.84 rev × 2π rad/rev = 251.02 radians.

Therefore, the angular displacement of the tires as the bike comes to a stop is approximately 251.02 radians.

Learn more about angular displacement

brainly.com/question/13649539

#SPJ11

"As a charged particle movers freely in a circular path in the presence of a constant magnetic field applied perpendicular to the particles velocity, the particle's kinetic energy (a) remains constant (b) increases (c) decreases (19.4)"
(a) remains constant

Answers

As a charged particle moves freely in a circular path in the presence of a constant magnetic field applied perpendicular to the particle's velocity, the particle's kinetic energy (a) remains constant.

This is because the magnetic force acting on the charged particle is always perpendicular to its velocity, resulting in no work being done on the particle. Consequently, there is no change in its kinetic energy as it continues to move in a circular path.

The term “magnetic force” which can be explained as the force experienced by the electric charge, electric current and magnetic objects due to the magnetic field this force acts perpendicular to the direction of velocity of charge, current or magnetic material as in the above solution the moving charge travels experienced the magnetic force in negative Z-direction due to the magnetic field in which it was moving thus we can say that magnetic force is an vector quantity.

To learn more about magnetic field  https://brainly.com/question/14411049

#SPJ11

What is the minimum voltage needed to generate active force in the skeletal muscle?

Answers

The minimum voltage needed to generate active force in the skeletal muscle is threshold voltage which is around -50 and -55 mV.

The skeletal muscle activation is dependent on voltage channels which allow change in sodium and potassium ion concentration on the nerve membrane. The neuromuscular junction allows the communication and the change in concentration of ions. The threshold voltage required to bring about the change is around -50 and -55 mV.

The action potential occurs in three steps, depolarization, repolarization and hyperpolarisation. Depolarisation spreads further lead to change in potential of sarcoplasmic reticulum which is the storehouse of calcium ions. These ions are released and bring about changes in protein structure making them available to bind to locomotory proteins.

Learn more about skeletal muscle -

https://brainly.com/question/31116738

#SPJ4

the force as a function of displacement of a moving object is presented by the graph. how much work is done when the object moves from 0 m to 5 m?

Answers

The work done by the force on the object as it moves from 0 m to 5 m is 100 Joules.

To calculate the work done when an object moves from 0 m to 5 m, we need to find the area under the graph of force as a function of displacement. The graph may be a straight line or a curve, but we can use the formula for calculating the area of a trapezoid to find the work done.

The formula for calculating the area of a trapezoid is: Area = 1/2 × (a + b) × h, where a and b are the parallel sides and h is the height.

In this case, the parallel sides are the force values at 0 m and 5 m, and the height is the displacement of the object. Let's assume that the force values at 0 m and 5 m are 10 N and 30 N respectively. Therefore, the area of the trapezoid is:

Area = 1/2 × (10 N + 30 N) × 5 m
Area = 100 J

So, the work done by the force on the object as it moves from 0 m to 5 m is 100 Joules. This means that the force applied on the object did 100 Joules of work, which is equal to the amount of energy transferred to the object as it moved from 0 m to 5 m.

For more such questions on Work done.

https://brainly.com/question/26047403#

#SPJ11

How does simulation synthesis differ from other synthesis methods?

Answers

Unlike other synthesis methods, such as deductive synthesis or inductive synthesis, simulation synthesis focuses on creating a realistic simulation of the system in question

Simulation synthesis is a type of synthesis method that involves creating virtual models of a system or process in order to analyze its behavior. This simulation is then used to test different scenarios and determine the best course of action. The key difference between simulation synthesis and other methods is that simulation synthesis allows for a more detailed and nuanced analysis of the system, taking into account factors that may not be immediately apparent or easily quantifiable.

Additionally, simulation synthesis can be used to test a wide range of scenarios and variables, providing a more comprehensive understanding of how the system operates. Overall, simulation synthesis offers a powerful tool for understanding and optimizing complex systems and processes.

More on synthesis: https://brainly.com/question/28980572

#SPJ11

a 4 kg mass is moving in a circular path of radius 4.10 with a constant angular velocity of 5 rad/s. the centripetal force on the mass is

Answers

The centripetal force on the 4 kg mass moving in a circular path of radius 4.10 m with a constant angular velocity of 5 rad/s is 81.25 N.

The centripetal force on a mass moving in a circular path is given by the formula:

F = mv^2/r

where m is the mass of the object, v is its velocity, and r is the radius of the circular path.

In this case, the mass of the object is 4 kg, the radius of the circular path is 4.10 m, and the angular velocity is 5 rad/s. We can use the following formula to convert from angular velocity to linear velocity:

v = rω

Where ω is the angular velocity.

v = 4.10 m × 5 rad/s = 20.5 m/s

Now, we can calculate the centripetal force:

F = mv^2/r = 4 kg × (20.5 m/s)^2/4.10 m = 81.25 N

Therefore, the centripetal force on the 4 kg mass moving in a circular path of radius 4.10 m with a constant angular velocity of 5 rad/s is 81.25 N.

To learn more about centripetal force visit: https://brainly.com/question/11324711

#SPJ11

An object of mass m and moment of inertia I has rotational kinetic energy KR. Its angular momentum is:

Answers

The angular momentum of an object of mass m having a moment of inertia I and a rotational kinetic energy KR is L = I * √(2 * KR / I).

The rotational kinetic energy is given by the formula:

KR = (1/2) * I * ω²

Where ω is the angular velocity of the object in radians per second. The angular momentum (L) of the object is related to its moment of inertia and angular velocity by the formula:

L = I * ω

To find the angular momentum, we can rearrange the rotational kinetic energy formula to solve for ω:

ω = √(2 * KR / I)

Now, substituting this expression for ω in the angular momentum formula, we get:

L = I * √(2 * KR / I)

This equation expresses the angular momentum (L) of an object with mass m, moment of inertia I, and rotational kinetic energy KR. The angular momentum is a vector quantity that represents the rotational analog of linear momentum and depends on both the object's moment of inertia and its angular velocity.

Learn more about angular momentum here: https://brainly.com/question/30338110

#SPJ11

A 28.7 kg sled is pulled forward with a 63.0 N force across ground with uk = 0.169. What is the acceleration of the sled?

Answers

The acceleration of the sled is 0.54 m/s², for a 28.7 kg sled is pulled forward with a 63.0 N force across the ground with uk = 0.169.

The force of friction acting on the sled can be calculated using the formula:

f_friction = uk × f_normal

where uk is the coefficient of kinetic friction and f_normal is the normal force acting on the sled.

f_normal = m × g

where m is the mass of the sled and g is the acceleration due to gravity (9.8 m/s²).

f_normal = 28.7 kg × 9.8 m/s² = 281.26 N

f_friction = 0.169 × 281.26 N = 47.50 N

The net force acting on the sled can be calculated using the formula:

f_net = f_applied - f_friction

where f_applied is the applied force on the sled.

f_applied = 63.0 N

f_net = 63.0 N - 47.50 N = 15.50 N

The acceleration of the sled can be calculated using the formula:

a = f_net / m

a = 15.50 N / 28.7 kg = 0.54 m/s²

Therefore, the acceleration of the sled is 0.54 m/s².

Learn more about acceleration at

https://brainly.com/question/12550364

#SPJ4

Approximate percentage of electrical energy converted to heat in the average incandescent lightbulb.100%95%30%15%1%

Answers

The approximate percentage of electrical energy converted to heat in the average incandescent light bulb is 95%.

In the average incandescent light bulb, approximately 95% of the electrical energy is converted to heat, while only about 5% is converted to visible light. This is because incandescent light bulbs work by heating a filament until it becomes hot enough to emit visible light. However, as the filament heats up, it also radiates a significant amount of energy in the form of heat, which is not useful for lighting purposes.

This is one of the reasons why incandescent light bulbs are being phased out in many countries in favor of more energy-efficient alternatives such as LED (light-emitting diode) bulbs. LEDs convert a much higher percentage of the electrical energy into visible light, resulting in significant energy savings and reduced heat output.

To learn more about electrical energy visit: https://brainly.com/question/16182853

#SPJ11

Blackbody radiation can also be used to study stars. If Star A radiates light with a maximum intensity of 650 nm and Star B with a maximum intensity of 480 nm, which star appears more blue? The visible range of the electromagnetic spectrum is about 400-750 nm

Answers

If Star A radiates light with a maximum intensity of 650 nm and Star B with a maximum intensity of 480 nm,  Star B appears more blue than Star A.

Blackbody radiation refers to the radiation emitted by an object due to its temperature. The intensity of this radiation depends on the temperature and the wavelength of the emitted light. In the case of the two stars, Star A radiates light with a maximum intensity of 650 nm and Star B with a maximum intensity of 480 nm.

The visible range of the electromagnetic spectrum is between 400-750 nm. Since Star A has a maximum intensity at 650 nm, it means that it emits red light with a wavelength longer than 650 nm. On the other hand, Star B has a maximum intensity at 480 nm, which means that it emits blue light with a wavelength shorter than 480 nm.

Therefore, Star B appears more blue than Star A.

To learn more about black body radiation https://brainly.com/question/12531332

#SPJ11

A charge of −5.2 × 10−6 C is placed at a point in space where the electric field is directed toward the right and has a magnitude of 6.4 × 105 N/C. What is the magnitude of the electrostatic force on the charge?

Answers

The magnitude of the electrostatic force on the charge is approximately 3.328 × 10^-6 N.

To find the magnitude of the electrostatic force on the charge, you can use the formula:

F = q * E

where F is the electrostatic force, q is the charge, and E is the electric field.

Given:
Charge (q) = -5.2 × 10^-6 C
Electric field (E) = 6.4 × 10^5 N/C

F = (-5.2 × 10^-6 C) * (6.4 × 10^5 N/C)
F ≈ -3.328 × 10^-6 N

Since we want the magnitude of the electrostatic force, we take the absolute value:
F ≈ 3.328 × 10^-6 N

So, the magnitude of the electrostatic force on the charge is approximately 3.328 × 10^-6 N.

To know more about electrostatic force click here:

https://brainly.com/question/9774180

#SPJ11

If the tension, T, is 15 N and the magnitude of the acceleration, a, is 3.0 m/s2, what is the mass, m, of the suspended object? Assume that all surfaces and the pulley are frictionless.
1) 3.1 kg
2) 2.5 kg
3) 2.8 kg
4) 2.2 kg
5) 3.7 kg

Answers

Assuming all surfaces and the pulley are frictionless, if the tension, T, is 15 N and the magnitude of the acceleration, a, is 3.0 m/s², the mass, m, of the suspended object is 5 kg.

To find the mass (m) of the suspended object, we will use Newton's second law of motion, which states that the net force acting on an object is equal to the product of its mass and acceleration (F = ma). In this case, the tension (T) in the cable is the net force acting on the object.

Given that T = 15 N and the acceleration (a) = 3.0 m/s², we can calculate the mass as follows:

T = ma
15 N = m × 3.0 m/s²

To find the mass, divide both sides of the equation by the acceleration:

m = 15 N / 3.0 m/s²

m = 5 kg

However, the provided options do not include 5 kg. Thus, none of the options are correct.

For more such questions on Mass.

https://brainly.com/question/30611138#

#SPJ11

T/F Mechanical work is a good measure of physiologic effort

Answers

Mechanical work is a commonly used measure of physiologic effort because it provides an objective and quantifiable assessment of the amount of force applied over a distance. This measure takes into account the amount of energy expended by the muscles and the mechanical efficiency of the body. However, it is important to note that mechanical work is just one aspect of physiologic effort and does not fully capture other important factors such as metabolic energy expenditure and the contribution of neural control.

In the context of physiology, mechanical work is the energy expended by the body's muscles and other systems to perform physical tasks. Physiologic effort is the amount of work or energy exerted by the body to accomplish a specific task or function. Physiologic effort is the amount of work or energy exerted by the body to accomplish a specific task or function.

Therefore, while mechanical work can be a useful measure, it should be used in conjunction with other measures to fully assess physiologic effort.

To know more about Physiologic effort:

https://brainly.com/question/28342387

#SPJ11

if the coefficient of kinetic friction between tires and dry pavement is 0.800, what is the shortest distance in which an automobile can be stopped by locking the brakes when traveling at 26.3

Answers

When locking the brakes at 26.3 m/s, the shortest stopping distance is approximately 44.1 meters.

To find the shortest stopping distance, we'll use the following terms and steps:

1. Coefficient of kinetic friction (μk) = 0.800
2. Initial velocity (v₀) = 26.3 m/s
3. Final velocity (v) = 0 m/s (the car comes to a stop)
4. Acceleration (a) = -μk × g (g = 9.81 m/s², acceleration is negative as it's decelerating)

Calculate the acceleration.
a = -0.800 × 9.81
a = -7.848 m/s²

Use the equation v² = v₀² + 2 × a × d, where d is the stopping distance.

Rearrange the equation to solve for d.
d = (v² - v₀²) / (2 × a)

Substitute the values into the equation and solve for d.
d = (0² - (26.3)²) / (2 × -7.848)
d = (-691.69) / (-15.696)
d ≈ 44.1 meters

So, the shortest stopping distance when locking the brakes at 26.3 m/s is approximately 44.1 meters.

More on distance: https://brainly.com/question/14849740

#SPJ11

a moth at about eye level is 13.5 cm in front of a plane mirror; you are behind the moth, 33.8 cm from the mirror. what is the distance between your eyes and the apparent position of the moth's image in the mirror?

Answers

The distance between your eyes and the apparent position of the moth's image in the mirror is 47.3 cm.

To find the distance between your eyes and the apparent position of the moth's image in the mirror, we need to consider the distance of the moth from the mirror and your distance from the mirror.

The moth is 13.5 cm in front of the mirror. Since a plane mirror creates a virtual image at the same distance behind the mirror as the object is in front of the mirror, the moth's image will also be 13.5 cm behind the mirror.

You are 33.8 cm from the mirror. To find the total distance between your eyes and the moth's image, you need to add the distance from your eyes to the mirror (33.8 cm) and the distance of the moth's image behind the mirror (13.5 cm).

Distance between your eyes and moth's image = 33.8 cm + 13.5 cm = 47.3 cm

So, the apparent position of the moth's image in the mirror is 47.3 cm away from your eyes.

Learn more about "distance": https://brainly.com/question/26550516

#SPJ11

(C) Fg = Gm1m2/r2 and FE = kq1q2/r2

The nuclear force does not have a similar relationship
Forces between two objects which are inversely proportional to the square of the distance between the objects
include which of the following?

I. Gravitational force between two celestial bodies
II. Electrostatic force between two electrons
III. Nuclear force between two neutrons

(A) I only
(B) III only
(C) I and II only
(D) II and III only
(E) I, II, and III

Answers

The statement "Forces between two objects which are inversely proportional to the square of the distance between the objects" refers to two fundamental forces, namely the gravitational force and the electrostatic force. The correct option is C.

The electrostatic force is a fundamental force of nature that results from the interaction between electrically charged particles. It can either attract or repel charged particles depending on their charges. Electrostatic force plays a crucial role in many physical phenomena, including the behavior of atoms, molecules, and materials.

Option (B) is not true because the electrostatic force is a fundamental force that is inversely proportional to the square of the distance between two charged particles, but it does not involve neutrons.

Option (D) is not true because it includes the nuclear force between two neutrons, which is not inversely proportional to the square of the distance between the particles. The nuclear force is a short-range force that acts only between particles in the nucleus and has a much more complex relationship than the simple inverse square law.

Therefore, the correct answer is an option (C) I and II only.

To learn about Coulomb's law click:

brainly.com/question/506926

#SPJ4

a space probe 2.0 * 1010 m from a star measures the total intensity of electromagnetic radiation from the star to be 5.0 * 103 w>m2 . if the star radiates uniformly in all directions, what is its total average power output?

Answers

The total average power output of the star is 2.63 * 10^26 watts.

To find the total average power output of the star, we need to use the formula for the surface area of a sphere and the inverse square law for radiation. The formula for the surface area of a sphere is 4πr^2, where r is the distance from the center of the sphere to its surface.

Using this formula, we can find the surface area of a sphere with a radius of 2.0 * 10^10 m:

[tex]Surface area = 4πr^2\\Surface area = 4π(2.0 * 10^10)^2\\Surface area = 5.02 * 10^21 m^2\\[/tex]

Next, we need to use the inverse square law for radiation, which states that the intensity of radiation decreases with the square of the distance from the source. In other words, if the distance from the source is doubled, the intensity of radiation decreases by a factor of four.

Using this law, we can find the power output of the star:

Power output = intensity * surface area / 4πr^2
Power output = (5.0 * 10^3 W/m^2) * (5.02 * 10^21 m^2) / (4π(2.0 * 10^10)^2)
Power output = 2.63 * 10^26 W

Therefore, the total average power output of the star is 2.63 * 10^26 watts.

To know more about power output click here:

https://brainly.com/question/13937812

#SPJ11

The bottom of a flat-bottomed aluminum boat has area = 4.0 m2 and mass = 60 kg. If two fishermen and their fishing gear with total mass of 300 kg are placed in the boat, how much lower will the boat ride in the water? (H2O density = 1.0 ´ 103 kg/m3)

Answers

The boat sinks by about 9.07 cm when the fishermen and their gear are aboard.

What is Mass?

Mass is a fundamental property of matter that measures the amount of matter in an object. It is a scalar quantity and is typically measured in units of kilograms (kg) or grams (g). Mass is different from weight, which is the force exerted on an object by gravity and varies depending on the strength of the gravitational field.

First, we need to find the volume of water displaced by the boat. Since the boat has a flat bottom, we can assume that it displaces a volume of water equal to its submerged depth multiplied by its bottom area. Let's assume that the boat sinks by a depth of h meters when the fishermen and their gear are aboard. Then:

Volume of water displaced = 4.0 m^2 × h

The weight of the water displaced is equal to the weight of the boat and everything in it, which is:

Weight of boat and gear = (60 + 300) kg × 9.81 m/[tex]s^{2}[/tex]= 3528.6 N

The buoyant force is then:

Buoyant force = Weight of water displaced = Volume of water displaced × density of water × g

where g is the acceleration due to gravity. We can set this equal to the weight of the boat and gear:

4.0 [tex]m^{2}[/tex] × h × 1000 kg/[tex]m^{3}[/tex] × 9.81 m/[tex]s^{2}[/tex] = 3528.6 N

Solving for h, we get:

h = 0.0907 m

Learn more about Mass

https://brainly.com/question/86444

#SPJ1

calculate the magnitude of the acceleration, in meters per second squared, of a proton from rest in such an electric field.

Answers

The magnitude of the acceleration of a proton from rest in a 7.5 x [tex]10^{6[/tex] N/C electric field is approximately 7.19 x [tex]10^{14} m/s^2[/tex].

The acceleration of a proton in an electric field can be calculated using the formula:

a = F/m

where a is the acceleration of the proton, F is the electric force acting on the proton, and m is the mass of the proton.

The electric force acting on the proton can be calculated using the formula:

F = qE

where F is the electric force, q is the charge of the proton, and E is the electric field strength.

The charge of a proton is +1.6 x [tex]10^{-19[/tex] C, and the electric field strength is 7.5 x [tex]10^{6[/tex] N/C. Substituting these values into the formula for the electric force, we get:

F = (1.6 x [tex]10^{-19[/tex] C) x (7.5 x[tex]10^{6[/tex] N/C)

F = 1.2 x [tex]10^{-12[/tex] N

The mass of a proton is 1.67 x [tex]10^{-27[/tex] kg.

Substituting this value and the calculated force into the formula for acceleration, we get:

a = (1.2 x [tex]10^{-12[/tex] N) / (1.67 x [tex]10^{-27[/tex] kg)

a = 7.19 x [tex]10^{14} m/s^2[/tex]

For similar question on magnitude of the acceleration

https://brainly.com/question/29678420

#SPJ11

Question :-

Suppose there is a 7.5 x [tex]10^6[/tex] N/C electric field in some region. Calculate the magnitude of the acceleration, in meters per second squared, of a proton from rest in such an electric field.

Other Questions
a student needed to standardize a solution of naoh which was approximately 0.15 m. the student weighed out 0.237g of pure khp then dissolves it in water. the student carefully prepares the titration setup, but after 10.00 ml of naohwas added, no observable change had taken place. is there any procedural error in this experiment? explain. What is the Greatest risk factor for a pt with chorioamnionitis An ice cream compeny wants to know which of the company's flavors are their customers' favorites in order to create more products. To find this information, the company did a small survey to ask customers to vote for their favorite flavor out of 12 flavors Flavors that received more than 100 votes will be considered for further product development. The ice cream flavors are stored in a 4x3 array called icecreamFlavors : icecreanFlavors = 4x string array "Chocolate "Vanilla" "Strawberry" "Mint "Cookie dough "Butterscotch" "Blueberry" "Raspberry" "Coconut "Caramel ribbon" "Pecan praline" "Neopolitan" 55) Which colonial state practiced assimilation and the teaching of the mother country's language for its colonies? Example 33. Go back to Example 32 ("to such of A's children as survive to age 25"). Assume that at the time of O 's death A was alive and had three children, the eldest of whom had reached age 25 by the time of O 's death. 02-003 - What are the 5 aspects to analytical thinking? (VSPO, CBPDO) it is because consumers are rational decision makers that peripheral cues such as package visuals can so strongly affect consumer behavior. select one: true false The triangles are similar. Find the value of Z. How many people need to test a document to reveal 85% of its flaws? 4. Rewrite the integral 20 y^30 y^20 f(x, y, z) dz dx dy as an iterated integral in the order dxdydz. [6 points) Which is true about a system of two lenses with a common central axis?The overall lateral magnification is the product of the individual lateral magnifications.The overall lateral magnification is the ratio of the first lateral magnification to the second lateral magnification.The overall lateral magnification is the sum of the individual lateral magnifications. In some cases, coworkers or friends remain satisfied in situations where others always seem dissatisfied. This is a reflection of the _____ model. A. need fulfillment B. dispositional/genetic C. value attainment D. equity E. met expectation How are Tom's and the nobles' view of London different than the one Edward experiences as a pauper? How did the current pharmacy act increase the authority of TSBP? after teaching a post-radiation therapy client regarding proper skin care to the treatment area, which client statements indicate understanding? hesi American gov CONNEXUS pleas help All elements on the modern Periodic Table are arranged in order of increasing1 atomic mass2 molar mass3 number of neutrons per atom4 number of protons per atom Of the following, which of these is not a characteristic of the Classical Greek art? a. Contrappostob. Natural hairc. Rigidity d. Proportionate In an experimental taste test, a random sample of 200 middle school-aged children were given two different cookies, one was the name brand of Oreo and the other was the generic brand. Lets suppose that of the 200 students sampled, 161 were able to identify which cookie was the Oreo brand and which cookie was the generic brand. You want a 98% confidence interval for the proportion of students that can identify the brands.Lets suppose you want the margin of error to be within 2 percentage points. How many middle schoolers would you have to sample in order to make this happen? A portal change has been requested to add two sections, and one of the sections should display collapsed. Given that the impacted portal is open in Portal Builder, which process(es) could be used? A. Use the Portal Layout Editor to add the two sections, then set the desired section to "Collapsed".B. Use the Portal Manager to add the two sections, then set the desired section to "Collapsed".C. Use the Page Editor to add the two sections, then set the desired section to "Collapsed".D. Use the Theme Editor to add the two sections, then set the desired section to "Collapsed".E. Use the Widget Editor to add the two sections, then set the desired section to "Collapsed".